Art of Problem Solving

2025 AMC 12A Problems/Problem 20

Problem

The base of the pentahedron shown below is a $13 \times 8$ rectangle, and its lateral faces are two isosceles triangles with base of length $8$ and congruent sides of length $13$, and two isosceles trapezoids with bases of length $7$ and $13$ and nonparallel sides of length $13$.

[asy] import graph3; size(200); real l = 13; real w = 8; real offset = (l - 7)/2;  // 3 real midy = w/2;  // 4 real h = 12; triple O1 = (0,0,0); triple O2 = (l,0,0); triple O3 = (l,w,0); triple O4 = (0,w,0); triple T1 = (offset, midy, h); triple T2 = (l - offset, midy, h); currentprojection=orthographic((-4,-6,3)); draw(O4--O1--O2, linewidth(1)); draw(O2--O3--O4, dashed + linewidth(1)); draw(O3--T2, dashed + linewidth(1)); draw(O1--T1, linewidth(1)); draw(O4--T1, linewidth(1)); draw(O2--T2, linewidth(1)); draw(T1--T2, linewidth(1)); label("13", (O1+O2)/2, 3*-Y);  // Bottom length label("13", (O2+T2)/2, 1.5*X); label("13", (O4+T1)/2, 2*-X); label("8", (O1+O4)/2, 2*-X);   // Width label("7", (T1+T2)/2, 1.5*Z);    // Top length [/asy]

What is the volume of the pentahedron?

$\textbf{(A) } 416 \qquad \textbf{(B) } 520 \qquad \textbf{(C) }  528  \qquad  \textbf{(D) } 676 \qquad \textbf{(E) } 832$

Solution 1 (Split Into Three Parts)

Notice that the triangular faces have a slant height of $\sqrt{13^2-4^2}=\sqrt{153}$ and that the height is therefore $\sqrt{153-(\frac{13-7}{2})^2} = 12$. Then we can split the pentahedron into a triangular prism and two pyramids. The pyramids each have a volume of $\frac{1}{3}(3)(8)(12) = 96$ and the prism has a volume of $\frac{1}{2}(8)(12)(7) = 336$. Thus the answer is $336+96 \cdot 2 = \boxed{\text{(C) } 528}$

~ Shadowleafy

Solution 2

Note that the height is $12$ from the previous method \n Note that as you go up, the length and width both decrease linearly and reach $0$ at the end


So the answer is $\int_0^{12} (8 - \tfrac{2}{3}x)(13 - \tfrac{1}{2}x) dx = \boxed{528}$

Solution (3D Pythagoras)

Let $H$ be the height of the solid. By 3D Pythagoras (recursion of 2D Pythagoras), \[\left( \frac{8}{2} \right)^2+\left( \frac{13-7}{2} \right)^2+H^2=13^2=4^2+3^2+H^2=5^2+H^2\] This gives the $5-12-13$ triple, so $H=12$. Continue as in other solutions.

~imosilver

Note

It just reminds about one of old aime problem {https://artofproblemsolving.com/wiki/index.php?title=1983_AIME_Problems/Problem_11 1983 AIME Problem 11}. ~DRA777

Video Solution 1 by OmegaLearn

https://youtu.be/WwMDpnuZrJ4

Video Solution by SpreadTheLove

https://www.youtube.com/watch?v=BadjXLma84M


See Also

2025 AMC 12A (ProblemsAnswer KeyResources)
Preceded by
Problem 19
Followed by
Problem 21
1 2 3 4 5 6 7 8 9 10 11 12 13 14 15 16 17 18 19 20 21 22 23 24 25
All AMC 12 Problems and Solutions

These problems are copyrighted © by the Mathematical Association of America.